Stay ahead of learning milestones! Enroll in a class over the summer!

G
Topic
First Poster
Last Poster
k a April Highlights and 2025 AoPS Online Class Information
jlacosta   0
Apr 2, 2025
Spring is in full swing and summer is right around the corner, what are your plans? At AoPS Online our schedule has new classes starting now through July, so be sure to keep your skills sharp and be prepared for the Fall school year! Check out the schedule of upcoming classes below.

WOOT early bird pricing is in effect, don’t miss out! If you took MathWOOT Level 2 last year, no worries, it is all new problems this year! Our Worldwide Online Olympiad Training program is for high school level competitors. AoPS designed these courses to help our top students get the deep focus they need to succeed in their specific competition goals. Check out the details at this link for all our WOOT programs in math, computer science, chemistry, and physics.

Looking for summer camps in math and language arts? Be sure to check out the video-based summer camps offered at the Virtual Campus that are 2- to 4-weeks in duration. There are middle and high school competition math camps as well as Math Beasts camps that review key topics coupled with fun explorations covering areas such as graph theory (Math Beasts Camp 6), cryptography (Math Beasts Camp 7-8), and topology (Math Beasts Camp 8-9)!

Be sure to mark your calendars for the following events:
[list][*]April 3rd (Webinar), 4pm PT/7:00pm ET, Learning with AoPS: Perspectives from a Parent, Math Camp Instructor, and University Professor
[*]April 8th (Math Jam), 4:30pm PT/7:30pm ET, 2025 MATHCOUNTS State Discussion
April 9th (Webinar), 4:00pm PT/7:00pm ET, Learn about Video-based Summer Camps at the Virtual Campus
[*]April 10th (Math Jam), 4:30pm PT/7:30pm ET, 2025 MathILy and MathILy-Er Math Jam: Multibackwards Numbers
[*]April 22nd (Webinar), 4:00pm PT/7:00pm ET, Competitive Programming at AoPS (USACO).[/list]
Our full course list for upcoming classes is below:
All classes run 7:30pm-8:45pm ET/4:30pm - 5:45pm PT unless otherwise noted.

Introductory: Grades 5-10

Prealgebra 1 Self-Paced

Prealgebra 1
Sunday, Apr 13 - Aug 10
Tuesday, May 13 - Aug 26
Thursday, May 29 - Sep 11
Sunday, Jun 15 - Oct 12
Monday, Jun 30 - Oct 20
Wednesday, Jul 16 - Oct 29

Prealgebra 2 Self-Paced

Prealgebra 2
Sunday, Apr 13 - Aug 10
Wednesday, May 7 - Aug 20
Monday, Jun 2 - Sep 22
Sunday, Jun 29 - Oct 26
Friday, Jul 25 - Nov 21

Introduction to Algebra A Self-Paced

Introduction to Algebra A
Monday, Apr 7 - Jul 28
Sunday, May 11 - Sep 14 (1:00 - 2:30 pm ET/10:00 - 11:30 am PT)
Wednesday, May 14 - Aug 27
Friday, May 30 - Sep 26
Monday, Jun 2 - Sep 22
Sunday, Jun 15 - Oct 12
Thursday, Jun 26 - Oct 9
Tuesday, Jul 15 - Oct 28

Introduction to Counting & Probability Self-Paced

Introduction to Counting & Probability
Wednesday, Apr 16 - Jul 2
Thursday, May 15 - Jul 31
Sunday, Jun 1 - Aug 24
Thursday, Jun 12 - Aug 28
Wednesday, Jul 9 - Sep 24
Sunday, Jul 27 - Oct 19

Introduction to Number Theory
Thursday, Apr 17 - Jul 3
Friday, May 9 - Aug 1
Wednesday, May 21 - Aug 6
Monday, Jun 9 - Aug 25
Sunday, Jun 15 - Sep 14
Tuesday, Jul 15 - Sep 30

Introduction to Algebra B Self-Paced

Introduction to Algebra B
Wednesday, Apr 16 - Jul 30
Tuesday, May 6 - Aug 19
Wednesday, Jun 4 - Sep 17
Sunday, Jun 22 - Oct 19
Friday, Jul 18 - Nov 14

Introduction to Geometry
Wednesday, Apr 23 - Oct 1
Sunday, May 11 - Nov 9
Tuesday, May 20 - Oct 28
Monday, Jun 16 - Dec 8
Friday, Jun 20 - Jan 9
Sunday, Jun 29 - Jan 11
Monday, Jul 14 - Jan 19

Intermediate: Grades 8-12

Intermediate Algebra
Monday, Apr 21 - Oct 13
Sunday, Jun 1 - Nov 23
Tuesday, Jun 10 - Nov 18
Wednesday, Jun 25 - Dec 10
Sunday, Jul 13 - Jan 18
Thursday, Jul 24 - Jan 22

Intermediate Counting & Probability
Wednesday, May 21 - Sep 17
Sunday, Jun 22 - Nov 2

Intermediate Number Theory
Friday, Apr 11 - Jun 27
Sunday, Jun 1 - Aug 24
Wednesday, Jun 18 - Sep 3

Precalculus
Wednesday, Apr 9 - Sep 3
Friday, May 16 - Oct 24
Sunday, Jun 1 - Nov 9
Monday, Jun 30 - Dec 8

Advanced: Grades 9-12

Olympiad Geometry
Tuesday, Jun 10 - Aug 26

Calculus
Tuesday, May 27 - Nov 11
Wednesday, Jun 25 - Dec 17

Group Theory
Thursday, Jun 12 - Sep 11

Contest Preparation: Grades 6-12

MATHCOUNTS/AMC 8 Basics
Wednesday, Apr 16 - Jul 2
Friday, May 23 - Aug 15
Monday, Jun 2 - Aug 18
Thursday, Jun 12 - Aug 28
Sunday, Jun 22 - Sep 21
Tues & Thurs, Jul 8 - Aug 14 (meets twice a week!)

MATHCOUNTS/AMC 8 Advanced
Friday, Apr 11 - Jun 27
Sunday, May 11 - Aug 10
Tuesday, May 27 - Aug 12
Wednesday, Jun 11 - Aug 27
Sunday, Jun 22 - Sep 21
Tues & Thurs, Jul 8 - Aug 14 (meets twice a week!)

AMC 10 Problem Series
Friday, May 9 - Aug 1
Sunday, Jun 1 - Aug 24
Thursday, Jun 12 - Aug 28
Tuesday, Jun 17 - Sep 2
Sunday, Jun 22 - Sep 21 (1:00 - 2:30 pm ET/10:00 - 11:30 am PT)
Monday, Jun 23 - Sep 15
Tues & Thurs, Jul 8 - Aug 14 (meets twice a week!)

AMC 10 Final Fives
Sunday, May 11 - Jun 8
Tuesday, May 27 - Jun 17
Monday, Jun 30 - Jul 21

AMC 12 Problem Series
Tuesday, May 27 - Aug 12
Thursday, Jun 12 - Aug 28
Sunday, Jun 22 - Sep 21
Wednesday, Aug 6 - Oct 22

AMC 12 Final Fives
Sunday, May 18 - Jun 15

F=ma Problem Series
Wednesday, Jun 11 - Aug 27

WOOT Programs
Visit the pages linked for full schedule details for each of these programs!


MathWOOT Level 1
MathWOOT Level 2
ChemWOOT
CodeWOOT
PhysicsWOOT

Programming

Introduction to Programming with Python
Thursday, May 22 - Aug 7
Sunday, Jun 15 - Sep 14 (1:00 - 2:30 pm ET/10:00 - 11:30 am PT)
Tuesday, Jun 17 - Sep 2
Monday, Jun 30 - Sep 22

Intermediate Programming with Python
Sunday, Jun 1 - Aug 24
Monday, Jun 30 - Sep 22

USACO Bronze Problem Series
Tuesday, May 13 - Jul 29
Sunday, Jun 22 - Sep 1

Physics

Introduction to Physics
Wednesday, May 21 - Aug 6
Sunday, Jun 15 - Sep 14
Monday, Jun 23 - Sep 15

Physics 1: Mechanics
Thursday, May 22 - Oct 30
Monday, Jun 23 - Dec 15

Relativity
Sat & Sun, Apr 26 - Apr 27 (4:00 - 7:00 pm ET/1:00 - 4:00pm PT)
Mon, Tue, Wed & Thurs, Jun 23 - Jun 26 (meets every day of the week!)
0 replies
jlacosta
Apr 2, 2025
0 replies
Connecting chaos in a grid
Assassino9931   3
N 7 minutes ago by dgrozev
Source: Bulgaria National Olympiad 2025, Day 1, Problem 2
Exactly \( n \) cells of an \( n \times n \) square grid are colored black, and the remaining cells are white. The cost of such a coloring is the minimum number of white cells that need to be recolored black so that from any black cell \( c_0 \), one can reach any other black cell \( c_k \) through a sequence \( c_0, c_1, \ldots, c_k \) of black cells where each consecutive pair \( c_i, c_{i+1} \) are adjacent (sharing a common side) for every \( i = 0, 1, \ldots, k-1 \). Let \( f(n) \) denote the maximum possible cost over all initial colorings with exactly \( n \) black cells. Determine a constant $\alpha$ such that
\[
\frac{1}{3}n^{\alpha} \leq f(n) \leq 3n^{\alpha}
\]for any $n\geq 100$.
3 replies
Assassino9931
Apr 8, 2025
dgrozev
7 minutes ago
Balanced Tournaments
anantmudgal09   7
N 8 minutes ago by Mathgloggers
Source: The 1st India-Iran Friendly Competition Problem 1
A league consists of $2024$ players. A round involves splitting the players into two different teams and having every member of one team play with every member of the other team. A round is called balanced if both teams have an equal number of players. A tournament consists of several rounds at the end of which any two players have played each other. The committee organised a tournament last year which consisted of $N$ rounds. Prove that the committee can organise a tournament this year with $N$ balanced rounds.

Proposed by Anant Mudgal and Navilarekallu Tejaswi
7 replies
anantmudgal09
Jun 12, 2024
Mathgloggers
8 minutes ago
2 var inequalities
sqing   1
N 9 minutes ago by sqing
Source: Own
Let $ a,b \in [0 ,1] . $ Prove that
$$  \frac{a}{ 1+a+b^2 }+\frac{b }{ 1+b+a^2 }\leq \frac{2}{3}$$$$  \frac{a}{ 1+a^2+b }+\frac{b }{ 1+b^2+a  }\leq \frac{2}{3}$$$$  \frac{a}{ 1+a^2+b }+\frac{b }{ 1+b^2+a  }+\frac{ab }{1+ab }\leq \frac{7}{6}$$$$  \frac{a}{ 1+a^2+b }+\frac{b }{ 1+b^2+a  }+\frac{ab }{2+ab }\leq1$$$$ \frac{a}{ 1+a^2+b }+\frac{b }{ 1+b^2+a  }+\frac{ab }{1+2ab }\leq1$$
1 reply
+1 w
sqing
16 minutes ago
sqing
9 minutes ago
nice [symmedians in a triangle, < ABM = < BAN]
grodij   10
N 13 minutes ago by Lemmas
Source: IMO Shortlist 2000, G5
The tangents at $B$ and $A$ to the circumcircle of an acute angled triangle $ABC$ meet the tangent at $C$ at $T$ and $U$ respectively. $AT$ meets $BC$ at $P$, and $Q$ is the midpoint of $AP$; $BU$ meets $CA$ at $R$, and $S$ is the midpoint of $BR$. Prove that $\angle ABQ=\angle BAS$. Determine, in terms of ratios of side lengths, the triangles for which this angle is a maximum.
10 replies
grodij
Nov 14, 2004
Lemmas
13 minutes ago
No more topics!
Mmmmmm...Tasty!
whatshisbucket   34
N Feb 25, 2025 by cursed_tangent1434
Source: 2017 ELMO #4
An integer $n>2$ is called tasty if for every ordered pair of positive integers $(a,b)$ with $a+b=n,$ at least one of $\frac{a}{b}$ and $\frac{b}{a}$ is a terminating decimal. Do there exist infinitely many tasty integers?

Proposed by Vincent Huang
34 replies
whatshisbucket
Jun 26, 2017
cursed_tangent1434
Feb 25, 2025
Mmmmmm...Tasty!
G H J
G H BBookmark kLocked kLocked NReply
Source: 2017 ELMO #4
The post below has been deleted. Click to close.
This post has been deleted. Click here to see post.
whatshisbucket
975 posts
#1 • 8 Y
Y by Davi-8191, naw.ngs, centslordm, megarnie, green_leaf, Adventure10, Mango247, Funcshun840
An integer $n>2$ is called tasty if for every ordered pair of positive integers $(a,b)$ with $a+b=n,$ at least one of $\frac{a}{b}$ and $\frac{b}{a}$ is a terminating decimal. Do there exist infinitely many tasty integers?

Proposed by Vincent Huang
Z K Y
The post below has been deleted. Click to close.
This post has been deleted. Click here to see post.
kk108
2649 posts
#2 • 4 Y
Y by megarnie, centslordm, Adventure10, Mango247
Well this is definitely tasty's tasty problem!
This post has been edited 1 time. Last edited by kk108, Jun 26, 2017, 7:25 AM
Z K Y
The post below has been deleted. Click to close.
This post has been deleted. Click here to see post.
babu2001
402 posts
#3 • 14 Y
Y by shinichiman, kk108, Tugsuu, vjdjmathaddict, JasperL, tarzanjunior, Anar24, OlympiadIneqByBruteForce, centslordm, denistusk, skyguy88, qwedsazxc, Adventure10, Mango247
The answer is definitely a big $\text{NO} $. Call a number "bitter" if and only if it is not "tasty".

$\text{Claim :} $ If a number $n$ is "bitter", then so are all its multiples.

$\text{Proof :} $ Let $(a, b) $ such that $a+b=n$ be a tuple which destroys the "tasty" nature of $n$. Then $(ka, kb) $ trivially destroys the "tasty" nature of any multiple $nk$. So the claim is proven. $\square$

For any primes $p\geq 5$, observe that $(p-3,3)$ or $(p-6,6)$ destroys the "tasty" nature of most primes $p$. If they do not, then $p-3$ is power of $2 $ and $p-6$ is a power of $5$. Then $(p-9,9)$ requires that $(p-9)$ is a power of $2 $. So we have powers of two with difference $6$, they are $(p-3),(p-9)$ which is not possible for primes $>11$. So the claim proves that any number divisible by a prime $>11$ is "bitter". For primes $2, 3, 5,7,11$ observe that $(3,13)$ destroys "tasty" nature of $2^4$, $(13,14)$ destroys the "tasty" nature of $3^3$, $(13,12)$ destroys the "tasty" nature of $5^2$, $(26,23)$ destroys the "tasty" nature of $7^2$ and $(60,61)$ destroys the "tasty" nature of $11^2$ so the claim shows that any tasty number is a divisor of $2^3\cdot 3^2\cdot 5\cdot 7\cdot 11=9240$ hence there are finitely many "tasty" numbers.

Note : Since we know that all "tasty" numbers divide $9240$ we can actually obtain all of them. They are $\{3,4,5,6,7,8,9,11,12,15,21\}$.

EDIT : Yikes!! I thought only one of $\frac{a} {b}$ and $\frac{b} {a} $ has to be non-terminating. Sorry, this does not change the solution much though. Made relevant changes.
This post has been edited 8 times. Last edited by babu2001, Jun 26, 2017, 11:44 AM
Z K Y
The post below has been deleted. Click to close.
This post has been deleted. Click here to see post.
SAUDITYA
250 posts
#4 • 3 Y
Y by centslordm, Adventure10, Mango247
No ,
We will show that there is no tasty number $n > 2^{2017}$.
See that a rational number written in lowest form has terminating decimal expansion if and only if its denominator is of form $2^a5^b$.
Now,
Assume that $\exists n > 2^{2017}$ such that $n$ is tasty!
See that "$\frac{a}{b}$ or $\frac{b}{a}$" is terminating if and only if "$\frac{n}{a}$ or $\frac{n}{b}$" is terminating(Here we have $a+b =n$!)
Now,
By Bertrand's Postulate there exists primes $p,q$ in between $\frac{n}{8} \ge p , q \ge \frac{n}{64}$
See that $n >> 64\sqrt{n} => \frac{n}{8} > p,q > \sqrt{n} > 5$.See that $pq >n => pq\nmid n$. So WLOG $=> p \nmid n$ and $n-4p > 4p $
Consider the pairs $(n-p,p) , (n-2p,2p), (n-3p,3p)$ and $(n-4p,4p)$
See that $\frac{n}{pk} \forall k  = (1,2,3,4)$ is never terminating because $p >5$ will always divide the denominator even when written in lowest form
Now,
See that $\frac{n}{n-kp} \forall k = (1,2,3,4)$ has to be terminating or else $n$ will be
Case 1. $n$ is even
See that $2 | n-2p => 2 \nmid n-p$ and $n-3p$ or else $2|p$ ,contradiction!
Now,
$n-p > 7p >35$ and $gcd(n,n-p) =1 => n-p = 5^a > 5 => a >0$
$n-3p >5p > 25 $ and $gcd(n,n-3p) = 1$ or $3=> n-3p = 5^c$ or $3.5^c > 15 => c>0$
$=> 5|n-p$ and $5|n-3p => 5|2p$,contradiction!

Case 2. $n$ is odd
see that $2| n-3p => 2 \nmid n-2p$ and $n-4p$ or else $2|p$ ,contradiction!
Now,
$n-2p > 6p >30$ and $gcd(n,n-2p) = 1 => 5|n-2p$
$n-4p > 4p >20$ and $gcd(n,n-4p) = 1 => 5|n-4p$
So $5|2p$, contradiction!

Hence $n$ is not tasty! , contradiction
Z K Y
The post below has been deleted. Click to close.
This post has been deleted. Click here to see post.
rmtf1111
698 posts
#5 • 4 Y
Y by kk108, centslordm, Adventure10, Mango247
I like to overkill-PNT, Mihailescu, LTE, Case-bashing aaand also not-so-well written
This post has been edited 1 time. Last edited by rmtf1111, Jun 26, 2017, 7:50 AM
Z K Y
The post below has been deleted. Click to close.
This post has been deleted. Click here to see post.
TheDarkPrince
3042 posts
#6 • 7 Y
Y by RMO17geek, Maths_Guy, centslordm, denistusk, Adventure10, Mogmog8, Mango247
We say $k$ is yummy if $k$ is of the form $2^i5^j$ for $i,j \in \mathbb{N}_{0}$.
We know that a fraction is terminating if and only if the denominator is yummy.
Let $S = \{1,2,..,n-1\}$
$n = a+b$, so $b = n-a$, where $a \in S$.
So a number $n$ is tasty if for every $a$, atleast one of $a$ and $n-a$ is yummy $a \in S$.

So for a number $n$ to be tasty we need atleast $\lceil\left(\frac{n-1}{2}\right)\rceil$ of the elements of $S$ is of yummy.(This condition is just a minimal condition not a strong one)

We know that the number of numbers in $S$ of the form $2^k$ and of the form $5^m$ are $[\log_2 n]$ and $[\log_5 n]$ respectively, where $[x]$ is the greatest integer less than $x$.

So $n$ is tasty if atleast, $$\lceil\frac{n-1}{2}\rceil \leq \text{The number of numbers in $S$ that are yummy} \leq ([\log_2 n] + 1)([\log_5 n] + 1) \leq (\log_2 n +1)(\log_5 n +1)$$Let $n = 2^k$ for $k \in \mathbb{R}^{+}$.
As $5>2^2$, $\frac{k}{2}=\frac{\log_2 n}{2}>\log_5 n $.

So, $ (\log_2 n +1)(\log_5 n +1) \leq (k+1)(k/2 + 1) = \frac{(k+1)(k+2)}{2}$
For $k\geq 6$, $\frac{(k+1)(k+2)}{2} \leq \frac{(2^k - 1)}{2} = \frac{n - 1}{2}$.

But we had $\frac{n-1}{2} \leq (\log_2 n +1)(\log_5 n +1)$.

So none of the numbers greater than $64$ are tasty.
So we have that there doesn't exist infinite tasty numbers.
Z K Y
The post below has been deleted. Click to close.
This post has been deleted. Click here to see post.
shinichiman
3212 posts
#7 • 5 Y
Y by Tawan, Anar24, centslordm, Adventure10, Mango247
We prove that there doesn't exist infinitely many tasty numbers. Assume the contrary, consider sufficient large tasty number $n$. Let $p_k$ be the smallest prime so that $p_k \nmid n$ and $p_k >5 \; (k \ge 3)$. Let $p_1<p_2< \ldots, p_{k-1}$ be all primes less than $p_k$ and $p_i \mid n$ for all $1 \le i \le k-1$. We have $n \ge p_1 \cdots p_{k-1} > 3^{k-2}(p_k+1)$ (since $2p_{k-1}>p_k$ according Bernard postulate and $p_i \ge 2$ for all $i \ge 2$).

Pick $a=p_k$ then $b=n-p_k$ so $\frac ba$ is not terminating decimal. Hence, $\frac ab$ must be or $b=n-p_k=2^x5^y$.
Pick $a=2p_k$ then similarly $b=n-2p_k=2^m5^l$. Pick $a=3p_k$ then similarly $b=n-3p_k=2^u5^v$.

If $x=0$ then $n$ is even and $y \ge 1$ so $n=p_k+5^y > 3^{k-2}(p_k+1)$. Thus, $5 \mid n-p_k$. Since $n$ is even then $u=0$ so $n=3p_k+5^v$ which follows $v \ge 1$. Thus, $5 \mid n-3p_k$. Hence, $5 \mid 2p_k$, a contradiction.

If $x \ge 1$ then $n$ is odd so $m=0$ so $l \ge 1$ to get $n=2p_k+5^l>3^{k-2}(p_k+1)$. Thus, $5 \mid n-2p_k$. Since $n$ is odd so $u \ge 1$. Since $5 \mid n-2p_k$ so $v=y=0$. Thus, $n=3p_k+2^u=2p_k+5^l=p_k+2^x$ which follows $p_k=5^l-2^u=2^{x-1}-2^{u-1}$. Thus, $u=1$ or $n=3p_k+2<3^{k-2}(p_k+1)$, a contradiction.

Thus, there doesn't exist infinitely many tasty number $n$.
Z K Y
The post below has been deleted. Click to close.
This post has been deleted. Click here to see post.
whatshisbucket
975 posts
#8 • 3 Y
Y by centslordm, Adventure10, Mango247
For those wondering, the complete set of tasty numbers is $\{3,4,5,6,7,8,9,11,12,15,21\}.$
Z K Y
The post below has been deleted. Click to close.
This post has been deleted. Click here to see post.
Ankoganit
3070 posts
#9 • 4 Y
Y by kk108, shinichiman, centslordm, Adventure10
The answer is $\textsc{No}$. We will show that $n$ can't be tasty for sufficiently large values of $n$.

First, we'll prove a lemma:

Lemma: $\varphi(n)\ge \sqrt{\frac{n}{2}}$ for all $n>1$.
Proof: Note that the inequality $p^{a-1}(p-1)\ge \sqrt{p^a}$ holds for all prime $p\ge 3$ and for positive integers $a$. Indeed, For $a>1$, this is simply $p^{\frac{a}{2}-1}(p-1)\ge 1$, which is true, and for $a=1$, this is $p-1\ge p^{\frac12}$, which is again true. Also, this is true for $p=2$ and $a>1$, as seen from the above proof. So if $n$ is odd or divisible by $4$, the above inequality can be multiplied for all prime power divisors of $n$ to yield $\varphi(n)\ge \sqrt{n}$, which is stronger than the desired result. For $n=2m$ with $m$ odd, note that $\varphi(n)=\varphi(2)\varphi(m)\ge 1\cdot\sqrt{m}=\sqrt{\tfrac{n}{2}}$, as desired. $\square$

Consider some large $n$. Consider the $\tfrac{\varphi (n)}{2}$ pairs of integers $(i,n-i)$ for $1\le i<\tfrac{n}{2}$, with $(i,n)=1$. For any such pair $(i,n-i)$, we need that one of $\tfrac{i}{n-i}$ and $\tfrac{n-i}{i}$ is terminating. Note that $\tfrac{i}{n-i}=\tfrac{n}{n-i}-1$ and $\tfrac{n-i}{i}=\tfrac{n}{i}-1$, so this is same as saying one of $\tfrac{n}{i}$ and $\tfrac{n}{n-i}$ is terminating. Since both the denominators are relatively prime with $n$, this implies at least one of $i$ and $n-i$ is a number of the form $2^a5^b$ for non-negative integers $a,b$. So there are at least $\tfrac{\varphi(n)}{2}$ numbers of the form $2^a5^b$ not exceeding $n$. But $a$ can't exceed $\log_2n$ and $5$ can't exceed $\log_5n$, so the total number of such numbers less than or equal to $n$ can't be bigger than $(\log_2 n+1)(\log_5n+1)$, which is smaller than $\tfrac{1}{2}\sqrt{\tfrac{n}{2}}$ and hence smaller than $\tfrac12\varphi(n)$ for large enough $n$, because asymptotics. This proves our claim. $\blacksquare$
Z K Y
The post below has been deleted. Click to close.
This post has been deleted. Click here to see post.
Ankoganit
3070 posts
#11 • 2 Y
Y by centslordm, Adventure10
TheDarkPrince wrote:
We know that a fraction is terminating if and only if the denominator is yummy.
That's not true. $\frac 36$ has a terminating decimal expansion but $6$ isn't yummy.
Z K Y
The post below has been deleted. Click to close.
This post has been deleted. Click here to see post.
kk108
2649 posts
#12 • 3 Y
Y by centslordm, Adventure10, Mango247
Yeah I was about to say that...
Z K Y
The post below has been deleted. Click to close.
This post has been deleted. Click here to see post.
vjdjmathaddict
502 posts
#13 • 3 Y
Y by centslordm, Adventure10, Mango247
perhaps the easiest problem on the test.
Z K Y
The post below has been deleted. Click to close.
This post has been deleted. Click here to see post.
TheDarkPrince
3042 posts
#14 • 4 Y
Y by Maths_Guy, centslordm, Adventure10, Mango247
Ankoganit wrote:
TheDarkPrince wrote:
We know that a fraction is terminating if and only if the denominator is yummy.
That's not true. $\frac 36$ has a terminating decimal expansion but $6$ isn't yummy.

Yup I agree, I made a mistake. :(
Z K Y
The post below has been deleted. Click to close.
This post has been deleted. Click here to see post.
MSTang
6012 posts
#15 • 4 Y
Y by Wave-Particle, centslordm, Adventure10, Mango247
We prove that all sufficiently large $n$ are not tasty.

Suppose $n > 2$ is tasty. Choose some integer $a \in \{1, 2, \ldots, n-1\}$ with $\gcd(a, n) = 1$; there are $\phi(n)$ such $a$. Defining $b = n-a$, note that $\gcd(b, n) = 1$, and either $\tfrac{a}{b} = \tfrac{n}{b} - 1$ or $\tfrac{b}{a} = \tfrac{n}{a} - 1$ terminates. Thus either $\tfrac{n}{a}$ or $\tfrac{n}{b}$ terminates. Since $\gcd(a, n) = \gcd(b, n) = 1$, either $a$ or $b$ is of the form $2^e5^f$ for some integers $e, f \ge 0$. Thus there are at least $\tfrac12\phi(n)$ integers in the interval $\{1, 2, \ldots, n-1\}$ of the form $2^e5^f$ for some $e,f \ge 0$. Note that \[2^e5^f \le n \implies 2^{e+f} \le n \implies e+f\le \log_2(n).\]Hence there are at most $\tfrac12 (\log_2(n) + 1)(\log_2(n) + 2)$ such pairs $(e,f)$. Thus we need \[\tfrac12 \phi(n) \le \tfrac12 (\log_2(n) + 1)(\log_2(n) + 2).\]But recall that \[\sum_{d \mid n} \phi(d) = n,\]and furthermore we have the fact $d \mid n \implies \phi(d) \mid \phi(n)$. Thus $\sum_{d \mid n} \phi(d) \le \sum_{d \mid n} \phi(n) = \tau(n)\phi(n)$, so $\phi(n) \ge \frac{n}{\tau(n)}$. Furthermore, $\tau(n) \le 2\sqrt{n}$ since divisors of $n$ come in pairs, one greater than $\sqrt{n}$ and one less than $\sqrt{n}$. Thus $\phi(n) \ge \tfrac12 \sqrt{n}$, so \[\sqrt{n} \le 2(\log_2(n)+1)(\log_2(n)+2)\]which fails for sufficiently large $n$. $\square$
Z K Y
The post below has been deleted. Click to close.
This post has been deleted. Click here to see post.
smy2012
688 posts
#16 • 3 Y
Y by centslordm, Adventure10, Mango247
My solution involves finding a class of "untasty" numbers.
That is for prime $p$,$p^l$ is not tasty if $p^{l-1}(p-1)>2\log_2 p\cdot \log_5 p $
Thus, exists a $N$ s.t. for any prime power $p^s>N$, $p^s$ is untasty. We can only have finitely many possible tasty number.
This post has been edited 1 time. Last edited by smy2012, Jun 29, 2017, 8:43 AM
Z K Y
The post below has been deleted. Click to close.
This post has been deleted. Click here to see post.
atmchallenge
980 posts
#17 • 3 Y
Y by centslordm, Adventure10, Mango247
Problem 4

Could someone look over my solution and let me know where I went wrong? I personally can't seem to find the error :( .
Z K Y
The post below has been deleted. Click to close.
This post has been deleted. Click here to see post.
tree3
641 posts
#18 • 9 Y
Y by Ankoganit, JustAnAverageAopser, vjdjmathaddict, atmchallenge, SD2014, Wizard_32, centslordm, Adventure10, Mango247
I think this is new.
Problem 4
This post has been edited 7 times. Last edited by tree3, Jul 3, 2017, 10:39 PM
Z K Y
The post below has been deleted. Click to close.
This post has been deleted. Click here to see post.
Superguy
354 posts
#20 • 3 Y
Y by centslordm, Adventure10, Mango247
So here is my lengthy solution.
Solution
This post has been edited 1 time. Last edited by Superguy, May 29, 2018, 4:31 AM
Z K Y
The post below has been deleted. Click to close.
This post has been deleted. Click here to see post.
aadhitya
98 posts
#21 • 2 Y
Y by centslordm, Adventure10
Not sure if my solution is correct.....

Let $n$ be a tasty number.
Let $d$ be a natural number less than $n$ and relatively prime to $n$. Also $d$ is not of the form $2^x.5^y$.

Notice that $\frac{n-d}{d}$ can't have a terminating decimal representation because $n-d$ and $d$ are co prime and d contains some prime factors other than $2$ and $5$.
So $\frac{p}{n-p}$ has a terminating decimal representation. This would imply that $n - p$ is of the form $2^x.5^y$.

So $\phi(n)$ $-X$ $\leq X$ where $X$ denotes the number of numbers less than $n$ that are of the form $2^x.5^y$.
$\implies \phi(n) \leq 2X$.

One can easily show that $X < (\log_2(n) +1) . (\log_5(n)+1)$ and $\phi(n) \geq \frac{\sqrt{n}}{\sqrt{2}}$.

So, we have $ \frac{\sqrt{n}}{\sqrt{2}} \leq (\log_2(n) +1) . (\log_5(n)+1)$. But clearly this is not true for large $n$ since square root function grows more rapidly than logarithmic function.

Therefore, the answer is NO
This post has been edited 1 time. Last edited by aadhitya, Feb 21, 2019, 6:58 AM
Z K Y
The post below has been deleted. Click to close.
This post has been deleted. Click here to see post.
NMN12
26 posts
#23 • 2 Y
Y by centslordm, Adventure10
Answer: No all numbers greater than 21 are not tasty.
Claim: If a number $m$ is not tasty then all $km$ for k natural are not tasty.
Proof: Consider the pair $(a,b)$ for which $m$ is not tasty then $(ka,kb)$ shows that $km$ is not tasty.
Claim: All numbers $n$ greater than 21 are not tasty.
Proof: 11 and 13 are not tasty. Then $n$ is not multiple of 11 nor 13. Consider the pairs $(11,n-11)$ and $(13,n-13)$ either one of them shows that $n$ is not tasty. Because if not both $n-11$ and $n-13$ will be of the form $2^x5^y$ and we will have an exponential equation, which can be solved by using mod 8 and mod 5
This post has been edited 4 times. Last edited by NMN12, Feb 11, 2020, 2:23 PM
Z K Y
The post below has been deleted. Click to close.
This post has been deleted. Click here to see post.
green_leaf
225 posts
#24 • 4 Y
Y by lilavati_2005, Polynom_Efendi, char2539, centslordm
First note that $2,3,5,7$ are tasty.
Let $p>7$ be a prime. We will prove that if $p$ is tasty, then $p=11$.
See that at least one of $\frac{p-3}{3}$ or $\frac{3}{p-3}$ has a terminating decimal expansion. $\frac{p-3}{3}$ is irreducible and also the denominator is not of the form $2^i5^j$ so the $\frac{3}{p-3}$ must be terminating. So $p-3=2^a5^b$.
Similarly, taking $(6, p-6)$ gives $p-6=2^c5^d$.
Finally, $p-7=2^e5^f$. So, $2^e5^f+4=2^a5^b$. So clearly, $f=b=0$ and $e=2, a=3$.
Hence, $p=8+3=11$. We can verify that $11$ is indeed tasty.

So, all tasty primes are in the set $S= \{2, 3, 5, 7, 11 \}$.
Now, suppose that some $n$ is not tasty.
So, there exists $a,b$ with $a+b=n$ and $\frac{a}{b}$ and $\frac{b}{a}$ are both irreducible.
Now, consider $kn$ for any natural $k$.
Clearly, if we choose $(ka, kb)$, we get $\frac{ka}{kb}=\frac{a}{b}, \frac{kb}{ka} = \frac{b}{a}$ which are clearly both irreducible by choice of $a,b,n$.

Hence any multiple of non-tasty integer is also non-tasty.
From this, it is clear that all tasty integers have only prime divisors in $S$.
Let a tasty integer $n=2^a3^b5^c7^d11^e$ where $n$ is sufficiently large.
Now consider $(13, n-13)$, we get $\frac{n-13}{13}$ is clearly irreducible and also non terminating.
Hence, for $n$ to be tasty, $\frac{13}{n-13}$ must be terminating.
So, $n-13=2^a3^b5^c7^d11^e-13=2^r5^s$.
Similarly, for $17$ we have $n-17=2^a3^b5^c7^d11^e-17=2^t5^u$. So $2^r5^s-4=2^t5^u$. Hence we can only have $r=3, s=0, t=2, u=0$.
So, $n-13=2^3=8 \implies n=21$ is the max value of $n$.
So, there are only finitely many values of $n$ and we are done.
Z K Y
The post below has been deleted. Click to close.
This post has been deleted. Click here to see post.
ducktility
20 posts
#25 • 1 Y
Y by centslordm
whatshisbucket wrote:
An integer $n>2$ is called tasty if for every ordered pair of positive integers $(a,b)$ with $a+b=n,$ at least one of $\frac{a}{b}$ and $\frac{b}{a}$ is a terminating decimal. Do there exist infinitely many tasty integers?

Proposed by Vincent Huang

Nice problem.

We consider the $\phi (n)$ numbers which are co-prime to $n$. Suppose $(a, b)$ are a pair of such numbers such that $a+b = n$. Then, either of the two can be expressed as $2^\alpha 5^\beta$. In total, there are at most $log _2(n)log _5 (n) + log _2(n) + log _5 (n) + 1$ such numbers. So, $(log_2(n) +1)(log_5(n)+1) \geq \frac{1}{2}\phi (n)$. Now, we use $\frac{1}{2}\phi (n) \geq \frac{1}{4}\sqrt{n}$ and get $ 4(log_2(n) +1)(log_5(n)+1) \geq \sqrt{n}$ which is not true for $n \geq 10^8$ (Not the best bound but it works and I know really really really huge bound but it does our job). Hence, the number of such tasty numbers is finite.
This post has been edited 3 times. Last edited by ducktility, Jul 13, 2020, 5:14 PM
Z K Y
The post below has been deleted. Click to close.
This post has been deleted. Click here to see post.
hakN
429 posts
#26 • 1 Y
Y by centslordm
Lets call the pairs $(a,b) , a + b = n$, with $a<b$ and $\gcd{(a,b)} = 1$ lucky. In a lucky pair $(a,b)$, we must have at least one of $a,b$ of the form $2^{x}\cdot 5^{y} , x,y \geq 0$. Now consider big enough $n$.
It is clear that there are $\frac{\varphi(n)}{2}$ lucky pairs. Also since $a,b < n$ and at least one of them must be of the form $2^x \cdot 5^y$, there are at most $(\log_2n + 1)(\log_5n + 1)$ such pairs.
So we must have $\frac{\varphi(n)}{2} \leq (\log_2n + 1)(\log_5n + 1) < (\log_2n + 1)^2$.
Now, using the inequality when $n\geq 7$, $\varphi(n) \geq \sqrt{n}$, we must have
$\frac{\sqrt{n}}{2} < (\log_2n + 1)^2$.
But that is clearly false for large enough $n$, and so there are finitely many $n$ satisfying the condition.
This post has been edited 2 times. Last edited by hakN, Jun 4, 2021, 5:34 PM
Reason: fixed typo
Z K Y
The post below has been deleted. Click to close.
This post has been deleted. Click here to see post.
mathlogician
1051 posts
#27
Y by
The answer is no. Define a number that can be expressed in the form $2^m5^n$ for positive integers $(m,n)$ as rare. Remark that if $n$ is tasty, then for each pair of relatively positive integers $(a,b)$ such that $a+b=n$, at least one of $a$ or $b$ must be rare. This implies that we must have at least $\tfrac{\varphi(n)}{2}$ rare numbers less than $n$. On the other hand, the number of rare integers less than $n$ is at most $(\log_2(n)+1)(\log_5(n)+1),$ so if $n$ is tasty, it must follow that $$(\log_2(n)+1)(\log_5(n)+1) \geq \frac{\varphi(n)}{2}.$$I claim that this inequality is false for all sufficiently large $n$. It fact, since logarithmic functions grow much slower than the square root function, it suffices to prove the following well-known claim:

Claim: $\varphi(n) \geq \tfrac{1}{2} \sqrt{n}$

Proof: We will prove this by strong induction. First, note that if $n$ is a power of $2$, $\varphi(n) = \tfrac{1}{2}n \geq  \tfrac{1}{2} \sqrt{n}.$ Otherwise, let $p$ be the greatest (odd) prime divisor of $n$, and write $n = p^ek$ for some positive integers $e$ and $k$. Then note that $$\varphi(n) = \varphi(p^e)\varphi(k) \geq \frac{\varphi(p^e) \sqrt{k}}{2} = \frac{p^{e-1}(p-1) \sqrt{k}}{2} \geq \frac{\sqrt{pk}}{2} = \frac{1}{2} \sqrt{n},$$as desired.
Z K Y
The post below has been deleted. Click to close.
This post has been deleted. Click here to see post.
CT17
1481 posts
#28
Y by
We claim the answer is no.

First, if $n = ab$ where $a$ is not tasty, then there is some pair $c+d=a$ such that neither $\frac{c}{d}$ nor $\frac{d}{c}$ is a terminating decimal. Writing $n = bc + bd$ implies $n$ is not tasty, so a number is not tasty if any of its divisors is not tasty.

For any prime $p > 13$, at least one of the pairs $(p-11,11)$, $(p-12,12)$, and $(p-13,13)$ implies that $p$ is not tasty.

$(6,7)$ implies that $13$ is not tasty.

$(3,118)$ implies that $121$ is not tasty.

$(3,46)$ implies that $49$ is not tasty.

$(3,22)$ implies that $25$ is not tasty.

$(13,14)$ implies that $27$ is not tasty.

$(3,13)$ implies that $16$ is not tasty.

Since only finitely many positive integers are multiples of none of the above numbers, we are done.
This post has been edited 1 time. Last edited by CT17, Nov 22, 2021, 10:00 PM
Z K Y
The post below has been deleted. Click to close.
This post has been deleted. Click here to see post.
megarnie
5596 posts
#29
Y by
We claim the answer is $\boxed{\text{no}}$.


Call an integer that isn't tasty disgusting.

Claim: If an integer $n$ is disgusting, then all multiples of $n$ are disgusting.
Proof: Suppose $a+b=n$ and $\frac{a}{b}$ and $\frac{b}{a}$ are repeating decimals. Obviously $\frac{ka}{kb}=\frac{a}{b}$ and $\frac{kb}{ka}=\frac{b}{a}$ are repeating for any positive integer $k$, which proves our claim.

Claim: All primes $p\ge 13$ are disgusting.
Proof: Let $P(a,b)$ denote the given assertion.

If $p\equiv 1\pmod 3$, then $P(p-7,7)$ works since $3|p-7$ and $7\nmid p-7$.

If $p\equiv 2\pmod3$, then $P(p-11,11)$ works since $3|p-11$ and $11\nmid p-11$.




Now we will find powers of $2,3,5,7,11$ which are disgusting.


$P(7,9)\implies 16$ is disgusting.

$P(13,14)\implies 27$ is disgusting.

$P(12,13)\implies 25$ is disgusting.

$P(12,37)\implies 49$ is disgusting.

$P(13,108)\implies 121$ is disgusting.


Thus, a tasty number must be a divisor of $2^3\cdot 3^2\cdot 5\cdot 7\cdot 11$, which solves the problem.


Next, we find all tasty integers.

If $n$ is the power of some prime, we get $n=\boxed{\{4,8,3,9,5,7,11\}}$ all work.


Now suppose two primes $p$ and $q$, with $p<q$ divide $n$.

Case 1: $p=2$.
Then by $P(3,7)$, we find $10$ is disgusting, so $q\ne5$.

By $P(3,11)$, $14$ is disgusting, so $q\ne7$.

By $P(9,13)$, $22$ is disgusting, so $q\ne11$.

Thus, $q=3$.

$P(7,11)\implies 18$ is disgusting.

So $\nu_3(n)=1$.

$P(11,13)\implies 24$ is disgusting.

Thus, our solutions here are $\boxed{\{6,12\}}$, which both work.



Case 2: $p=3$.
Then $P(13,20)\implies 33$ is disgusting.

Thus, $q\in\{5,7\}$.

$P(11,34)\implies 3^2\cdot 5$ is disgusting.

$P(11,52)\implies 3^2\cdot 7$ is disgusting.

So $\nu_3(n)=1$, which gives $\boxed{\{15,21\}}$, which both work.


Case 3: $p>3$.
$P(13,22)\implies 5\cdot 7$ is disgusting.

$P(13,42)\implies 5\cdot 11$ is disgusting.

$P(19,58)\implies 7\cdot 11$ is disgusting.

So there are no solutions for this case.



Thus, our solution set in increasing order is $\{3,4,5,6,7,8,9,11,12,15,21\}$.
Z K Y
The post below has been deleted. Click to close.
This post has been deleted. Click here to see post.
guptaamitu1
656 posts
#30
Y by
The answer is NO. We will show all sufficiently large $n$ are not tasty. We look at the ordered pairs when $\gcd(a,b) = 1$. This equals $\phi(n)$. Let $M$ satisfy $2^M \le n < 2^{M+1}$. Observe $n$ has at most $M$ distinct prime divisors. If we denote by $p_i$ the $i$-th prime, then
$$\phi(n) \ge n \cdot  \prod_{i=1}^M \left(1 - \frac{1}{p_i} \right) \ge n \cdot \prod_{i=1}^M \left( 1 - \frac{1}{i+1} \right) = n \cdot \frac{1}{M+1} = \frac{n}{M+1} \ge \frac{2^M}{M+1}$$Call a number \emph{good} if $2,5$ are its only prime divisors. Now note when $\gcd(a,b) = 1$, then for $n$ to be tasty $a,b$ must be good. Observe any numbers $\le n$ has $v_2,v_5 < M+1$. Hence number of good numbers $\le n$ is at most $(M+1)^2$. So number of ordered pairs $(a,b)$ with $\gcd(a,b) = 1$ is at most $(M+1)^4$. This forces the inequality
$$\frac{2^M}{M+1} \le (M+1)^4$$Since this cannot hold for large $M$, so all large enough $n$ are not tasty, as desired. $\blacksquare$
Z K Y
The post below has been deleted. Click to close.
This post has been deleted. Click here to see post.
BarisKoyuncu
577 posts
#31 • 1 Y
Y by SerdarBozdag
Here is a different solution.

First of all, see that if a number is not tasty, then so are all its multiples.

Now, check that $13,17,19$ is not tasty since $13=6+7, 17=6+11, 19=6+13$.

Choose an integer $n>22$. If $n$ is divisible by any of $13,17,19$; then we are done. Assume that $(n,13\cdot 17\cdot 19)=1$.

Now consider the pairs $(13,n-13), (17,n-17)$ and $(19,n-19)$. Since these pairs are relatively prime, all the numbers $n-13, n-17, n-19$ should be in the form $2^x\cdot 5^y$.
See that at most one of them is divisible by $5$. Hence at least two of them is a power of $2$. But, observe that this implies $n=22$. We assumed $n\ge 23$, contradiction. Hence, none of the numbers greater than $22$ is tasty.
This post has been edited 1 time. Last edited by BarisKoyuncu, Apr 12, 2022, 5:29 AM
Z K Y
The post below has been deleted. Click to close.
This post has been deleted. Click here to see post.
ricegang67
26 posts
#32
Y by
The answer is no.

Claim. If $n$ is tasty, then any factor of $n$ is also tasty.

Proof. Let $d > 2$ be a factor of some tasty $n$. Then, for all choices of $a$ and $b$ such that
\[\frac{n}{d}a + \frac{n}{d}b = n,\]we must have $a + b = d$ and $\frac{na/d}{nb/d} = \frac{a}{b}$ is terminating. Thus for all $(a, b)$ with $a + b = d$, $\frac{a}{b}$ terminates as required. $\square$

Now, it suffices to show that there are only finitely many tasty prime powers. First, we will show that any $p^k$ with $p \ge 13$ is not tasty. This is simple: consider the pairs $(6, p - 6)$ and $(12, p - 12)$. Clearly $\gcd(6, p - 6) = \gcd(12, p - 12) = 1$, so if $p$ is tasty, then $p - 6$ and $p - 12$ must both be of the form $2^x 5^y$. However, $p - 6$ and $p - 12$ are odd, so they are powers of 5, but there are no powers of 5 that differ by 6, so this is impossible.

We therefore only need to show that there are finitely many powers of 2, 3, 5, 7, and 11 which are tasty. Applying identical logic to above on the pairs $(17, p^{k} - 17)$ and $(19, p^{k} - 19)$ shows that this is the case.
This post has been edited 1 time. Last edited by ricegang67, Feb 21, 2023, 9:56 PM
Reason: make sol more concise
Z K Y
The post below has been deleted. Click to close.
This post has been deleted. Click here to see post.
peppapig_
281 posts
#33 • 1 Y
Y by mulberrykid
We claim that all integers greater than $1323$ cannot be tasty. We can take this in cases.

Case 1: $\nu_3{(n)}>2$
$27$ itself is not tasty, so for any multiple of $27$, $27k$ is also not tasty because $\frac{13k}{14k}=\frac{13}{14}$, of which neither itself nor its reciprocal are terminating decimals.

Case 2: $\nu_7{(n)}>1$
Similarly, $49$ is not tasty ($49=3+46$), therefore no multiple of $49$ is tasty.

Case 3: $\nu_3{(n)}=0$, $\nu_7{(n)}=0$
Since all numbers greater than $21$ relatively prime to both $3$ and $7$, $n$ can be expressed as $3x+7y$ for some $x$ and $y$, $n$ cannot be tasty. This is because when simplified, one side will be a multiple of $7$ and the other will be a multiple of $3$. Therefore no numbers in this case greater than $1323$ are tasty.

Case 4: $\nu_3{(n)}=1$, $\nu_7{(n)}=0$
Similarly, every $n>63$ in this case can be expressed as $9x+21y$ for some $x$, $y$, making $n$ not tasty.

Case 5: $\nu_3{(n)}=2$, $\nu_7{(n)}=0$
Similarly, every $n>189$ in this case can be expressed as $27x+63y$ for some $x$, $y$, making $n$ not tasty.

Case 6: $\nu_3{(n)}=0$, $\nu_7{(n)}=1$
Similarly, every $n>147$ in this case can be expressed as $3x+49y$ for some $x$, $y$, making $n$ not tasty.

Case 7: $\nu_3{(n)}=1$, $\nu_7{(n)}=1$
Similarly, every $n>441$ in this case can be expressed as $9x+147y$ for some $x$, $y$, making $n$ not tasty.

Case 8: $\nu_3{(n)}=2$, $\nu_7{(n)}=1$
Similarly, every $n>1323$ in this case can be expressed as $27x+441y$ for some $x$, $y$, making $n$ not tasty.

Therefore no $n>1323$ can be tasty, proving that there are not infinitely many tasty integers, and we are done.
This post has been edited 3 times. Last edited by peppapig_, Mar 16, 2023, 2:23 PM
Reason: grrrr
Z K Y
The post below has been deleted. Click to close.
This post has been deleted. Click here to see post.
blackbluecar
302 posts
#34
Y by
There are finitely many tasty numbers. Note that $a/b$ is terminating if and only if $b$ is not divisible by any primes other than $2$ and $5$ given that $\gcd(a,b)=1$. We call a positive integer not divisible by any primes other than $2$ and $5$ stringy

Claim: If $F(n)$ denotes the number of stringy positive integers not exceeding $n$ then $\varphi(n) > 10^{100} \cdot F(n)$ for all sufficiently large $n$.

Indeed, note that for all large $n$ we have \[ \varphi(n) > \sqrt{\frac{n}{2}} > 10^{100} \cdot \log_2(n) \cdot \log_5(n) > 10^{100} \cdot F(n)\]as desired. $\square$

Note that $\varphi(n)/2$ denotes the number of ways to pick $a$ and $b$ so that $a+b=n$ and $\gcd(a,b)=1$. Since $\varphi(n)/2$ exceeds $F(n)$ for all large $n$, we may choose $\gcd(a,b)=1$ obeying $a+b=n$ and $b$ not only having prime divisors $2$ and $5$. $\blacksquare$
Z K Y
The post below has been deleted. Click to close.
This post has been deleted. Click here to see post.
HamstPan38825
8857 posts
#35 • 1 Y
Y by MihaiT
The answer is no; there are finitely many. Take $n$ sufficiently large and consider pairs $(a, b)$ with $a+b=n$ where $\gcd(a, n) = 1$.

Note that for $a/b$ or $b/a$ to be terminating among these pairs, one of $a, b$ must be of the form $2^k \cdot 5^\ell$. Since each such value for $a$ corresponds to a unique pair, the number $P$ of pairs must be loosely bounded above by $\log_2 n \cdot \log_5 n$. On the other hand, we have $$\sqrt n < \phi(n) = P < \log_2 n \cdot \log_5 n$$which is an obvious contradiction for $n$ large.

(It is well-known that $\phi(n) > \sqrt n$ for $n \neq 2, 6$; this can quickly be checked by multiplicativity of $\phi$.)
Z K Y
The post below has been deleted. Click to close.
This post has been deleted. Click here to see post.
N3bula
263 posts
#36
Y by
Note that if $m$ is not tasty any multiple of $m$ is not tasty. If a prime $p>21$ we get that there is a multiple of $3$ such that $3n<p$ and $7\mid 3n-p$ which means that if a value is
tasty it can only be divisible by primes smaller than $21$, if a value is only divisible by primes smaller than $21$ call this value $n$ and has the property $n>23\dot 29$,
we get that there is a multiple of 23 such that $23m<n$ and $29\mid n-23m$ which suffices to prove there are only finitely many tasty values.
Z K Y
The post below has been deleted. Click to close.
This post has been deleted. Click here to see post.
pie854
243 posts
#37
Y by
Let $n$ be sufficiently large and suppose that it is tasty. Let $a$ be such that $\gcd(a,n)=1$. Then, $\gcd(n-a,a)=1$. Since either $a/(n-a)$ or $(n-a)/a$ must be terminating, we must have that either $a$ or $n-a$ must be of the form $2^a 5^b$. Thus, the number of integers of the form $2^a 5^b$ less than $n$ must be greater than $\phi(n)/2$. But, this is obviously wrong for large enough $n$.
Z K Y
The post below has been deleted. Click to close.
This post has been deleted. Click here to see post.
cursed_tangent1434
600 posts
#38
Y by
We claim that the answer is in fact no. It is well known that the fraction $\frac{a}{b}$ is a terminating decimal if and only if the denominator $b$ is of the form $b=2^k\cdot 5^l$ for non negative integers $k$ and $l$.

Now, consider sufficiently large $n$. We shall show that it is not tasty. Since $n=(n-3)+3$ we must have $n-3=2^u5^v$ and since $n=(n-7)+7$ we must have $n-7=2^x5^y$. Plugging in one expression for $n$ into the other we obtain the diophantine,
\[2^x5^y+4=2^u5^v\]We have two cases.

Case 1 : If $y>0$, then the left hand side is clearly $4 \pmod{5}$ which implies that $v=0$. The equation then rewrites to,
\[2^x5^y+4=2^u\]for sufficiently large $n$ we must have $u>3$ so,
\[2^{x-2}5^y +1=2^{u-2}\]But then clearly the left hand side is odd and the right hand side is even unless $x=2$. Then,
\[5^y+1=2^{u-2}\]which has no solutions by Mihailescu's Theorem.

Case 2 : If $y=0$ then,
\[2^x +4=2^u5^v\]once again for sufficiently large $n$ we are required to have $x>3$ so,
\[2^{x-2}+1=2^{u-2}5^v\]But if $u>2$ the right hand side is even while the left hand side is odd which is a clear contradiction. Thus, $u=2$ and we have,
\[2^{x-2}+1=5^{v}\]which once again has no solutions when $v>1$ by Mihailescu's Theorem. But for sufficiently large $n$ we must have $v>1$ so this case fails to generate infinite solutions as well.

Thus, it is clear that the number of tasty integers is in fact finite, as desired.
Z K Y
N Quick Reply
G
H
=
a